LSAT and Law School Admissions Forum

Get expert LSAT preparation and law school admissions advice from PowerScore Test Preparation.

 biskam
  • Posts: 124
  • Joined: Aug 18, 2017
|
#40549
I'm confused as to why B is wrong and D is right. I thought B was preferable and a stronger principle overall because it included the idea encapsulated in the last sentence of the stimulus, meaning people should gain the greatest benefit possible. Is D right because the question is only asking us to focus on a principle that responds to the second sentence in the stimulus?

Thanks!
User avatar
 Dave Killoran
PowerScore Staff
  • PowerScore Staff
  • Posts: 5852
  • Joined: Mar 25, 2011
|
#40835
biskam wrote:I'm confused as to why B is wrong and D is right. I thought B was preferable and a stronger principle overall because it included the idea encapsulated in the last sentence of the stimulus, meaning people should gain the greatest benefit possible. Is D right because the question is only asking us to focus on a principle that responds to the second sentence in the stimulus?

Thanks!
Hi Biskam,

Thanks for the question! Let's start by looking at how answer choice (B) and the argument relate, because I'm noticing in your questions that you are at times misreading or misunderstanding the stimulus (I saw it in another question of yours that I answered earlier today). The last sentence of the stimulus states that, "Besides, if a masterpiece is to be fully appreciated, it must be seen alongside other works that provide a social and historical context for it." Answer choice (B) says, "In providing facilities to the public, the goal should be to ensure that the greatest possible number of people gain the greatest benefit possible from them." So, comparing those two, I don't see that (B) encapsulates the idea in the last sentence. The last sentence indicates that you need to see art in context to appreciate it whereas (B) says the goal is to provide the greatest benefit to the maximum number of people. Those are not the same idea, and one of the keys to knocking out wrong answers is seeing exactly what they say when compared to the stimulus; don't mentally fix an answer and "make it fit" the situation when it doesn't actually do so.

With answer choice (D), the first question is, what is the argument used in the stimulus against the critics? It's that the art critics' claim is unreasonable ("is in principle unwarranted") because there aren't enough masterpieces to distribute them widely (which is the same as saying that you can't practically distribute them as suggested by the critics) and that anyways you need to see them together to best understand them (so even if you could spread them widely, you wouldn't want to). In other words, the primary criticism is that you can't do what the critics are saying, and thus their criticism is unwarranted:

  • Premise: There aren't enough masterpieces to distribute them widely

    Conclusion: The art critics' claim is unreasonable

Thus, answer choice (D) matches with this well and helps strengthen the author's position, because if it is true (and we are told to accept it), then since there isn't a better system of displaying art work, we can conclude that the criticism is not reasonable. There's actually a smooth conditional relationship at the heart of this, as follows:

  • reasonable to criticize an arrangement as unfair :arrow: must be a more equitable arrangement that is practically attainable

But, we already know there's not a more equitable arrangement per the author's statement, and thus, with the necessary condition unfulfilled, we can properly conclude that the criticism should not have occurred. Answer choice (D) thus fills in the connection (via a contrapositive) between the author's premise and conclusion, and when seen altogether, the relationship appears as follows:

  • Premise/Ans. Choice (D): reasonable to criticize an arrangement as unfair :arrow: must be a more equitable arrangement that is practically attainable

    Premise: There aren't enough masterpieces to distribute them widely = more equitable arrangement that is practically attainable


    Conclusion: The art critics' claim is unreasonable = reasonable to criticize an arrangement as unfair

Please let me know if that helps. Thanks!
User avatar
 fortunateking
  • Posts: 31
  • Joined: Jan 10, 2022
|
#93643
That helps a lot thx!

Get the most out of your LSAT Prep Plus subscription.

Analyze and track your performance with our Testing and Analytics Package.